You are on page 1of 45

CHAPTER 2 – Solutions to Assignment Problems

Assignment 2.1:
a.
Freida, Incorporated
Income Statement
For the Year ending December 31, 2006

Gross sales 462,720


Less Returns and allowances (10,210)
Net sales 452,510

Beginning inventory 63,210


+ Materials Purchases 228,580
- Ending inventory (68,390)
Cost of goods sold 223,400
Gross profit 229,110
Management salaries 17,950
Lease payments 39,270
Advertising expenditures 12,930
R&D expenditures 4,890
Repairs and maintenance costs 2,910
Depreciation 12,510
Operating profit 138,650
Interest expense 17,090
Earnings before taxes 121,560
Taxes 3,270
Net income 118,290

b. Net profit margin = net income/net sales = 118,290/452,510 = 26.14%


c. Accumulated depreciation = 212,820 + 12,510 = $225,330.
Assignment 2.2:
a.

Windcharter Company
Balance Sheet
For the Year ending December 31, 2006

Cash 17,600 ST bank loans 32,570


Accounts receivable, net 105,770 Accounts payable 50,830
Inventories 136,500 Accrued expenses 11,850
Current assets 259,870 Current portion LT Debt 4,080
Current liabilities 99,330
Gross fixed assets 284,950 Long-term debt 134,300
Less Accumulated depreciation (82,310) Total liabilities 233,630
Net fixed assets 202,640 Preferred stock 8,000
Common stock ($0.20 par) 60,000
Additional paid in capital 71,600
Retained earnings 89,280
Total assets 462,510 Total liabilities & net worth 462,510

b. Assume for this problem that the number given for Net income is actually Net income
available to common stockholders (that is, reported Net income minus preferred dividends).
Thus, Annual addition to Retained earnings = Net income available to common stockholders
– Common stock dividends paid.
Thus, Common stock dividends paid = Net income – Addition to (i.e., change in) Retained
earnings.
Common stock dividends paid = 25,400 – (89,280 – 79,880) = 16,000.

Dividends per share = Dividends divided by number of shares outstanding.


Number of shares outstanding = Common Stock divided by par value per share
= 60,000/.20 = 300,000.

Dividends per share = $0.053 or 5.3 cents per share.

c. Cash spent on new plant and equipment = Depreciation for the year as listed on the income
statement plus the change in net fixed assets.
Thus, Cash spent on fixed assets in 1999 = 10,260 + (202,640 – 184660) = $28,240.
Assignment 2.3:
1. $4,055,740
$4,100,144
Although Gross sales decreased by 2 percent, net sales (the only number reported) show an
increase of 1.1 percent. Since Returns and allowances are estimates made by management,
there is some chance that the growth in sales conclusion may be misleading.

2. Cost of goods sold = Beginning inventory + Purchases – Ending inventory.


a. $174,300
b. $218,500
3. First solve for accumulated depreciation:
2005 2006
Gross fixed assets 3,200,000 4,620,000
Less Accumulated depreciation (1,280,000) (1,540,000)
Net fixed assets 1,920,000 3,080,000

Depreciation expense = change in accumulated depreciation = $260,000.


CHAPTER 2 – Answers to Additional Problems and Questions
Problems:
1. Look up definitions in chapter
2. EBIT = $15,000; Net income = $6,500
3. a. $400 b. $100 c. $35
4. Gross profit = $3,120,000; Ending Inventory = 850,000.
5.

Gross Sales 5,000,000


-Returns 100,000
Net Sales 4,900,000
-COGS 3,000,000 (250,000 x $12 per unit)
Gross profit 1,900,000
- Operating expenses 1,000,000
- Depreciation 100,000
Operating income 800,000
- Interest 500,000
EBT 300,000
- Taxes 120,000
Net income 180,000

6.
Sales 32,000
Cost of goods sold 19,200
Gross profit 12,800
Operating expenses 4,000
Depreciation 3,000
Operating profit 5,800
Interest expense 2,800
EBT 3,000
Taxes 900
Net income 2,100

7.
Inventory – current asset Retained earnings - equity
Accounts receivable – current asset Accounts payable – current liability
Long-term debt – noncurrent liability Accrued wages and taxes – current liability
Common stock (par value) - equity Notes payable (bank loans) – current liability
Plant and equipment – noncurrent asset Marketable securities – current asset
Cash – current asset Prepaid expenses – current asset

8. Total shareholders’ equity = $1,450,000.


9. In the following solution, the reported number for Gross fixed assets is assumed to exclude
Leasehold improvements.

Quest-Mar, Incorporated
Balance Sheet
For the Year Ending Decemebr 31, 2006

Cash 120 ST Bank loan 20


Net Accounts receivable 100 Accounts payable 90
Inventories 190 Accrued expenses 40
Current assets 410 Current portion LT Debt 60
Gross fixed assets (exc. Lease. Imp.) 900 Current liabilities 210
Leasehold improvements 300 Long-term bank loan 600
Less Accumulated depreciation (200) Total liabilities 810
Net fixed assets 1,000 Common stock ($1.00 par) 400
Retained earnings 200
Total stockholder's equity 600

Total assets 1,410 Total liabilities & net worth 1,410

10. a. $782,200 b. $54,532,500 c. $44,465,000

2004 2005 2006


11. CP LT Debt 280,000 280,000 280,000
LT Debt 2,240,000 1,960,000 1,680,000

12. a. 75,000 shares (note that all figures for this problem are in thousands).
b. (10,450,000 – 8,700,000) / 75,000 = $23.33 c. $1.12 (total dividends paid by the
company = NI(2006) – change in retained earnings = 700,000. Divide this by 625,000
shares outstanding to get $1.12 per share).
13. Total dividends paid = $6,000,000 ; Dividends per share = $1.20
14. a. EPS=$6.14; DPS = $1.89 b. $180,400 c. EPS=$6.01; DPS=$1.76 (assumes
3,000 shares issued – typo on problem).

15. a. 5.25 b. Price=8.25 c. 1.57 times


Equity represents shareholder wealth. Book value is historical. Market value is a function of
the expected future cash flows of the company. Book value is approximately what the firm’s
assets are worth if liquidated. Market value is what the company is worth as a functioning
entity. Thus, this company is worth approximately 1.57 times more as an operating company
than if the company stopped operating and sold off all of its assets. The larger this ratio, the
more successful the company is at creating value in operations.
16. a. Owners equity in 2005 = $140,000; owners equity in 2006 = $144,000
b. $10,000
c. $40,000
d. $4,000
e. $50,000
17. a. Owners equity in 2005 = $100,000; owners equity in 2006 = $110,000
b. $20,000; $24,000
c. $10,000
d. $3,000
e. $128,000 – note this merely equals the change in net fixed assets + depreciation from I/S
f. +$22,000
CHAPTER 3 – Solutions to Assignment Problems

Assignment 3.1:

Net income 48,000


+ Depreciation 30,000
+ Decrease in Accounts receivable 21,000
- Increase in Inventories (10,900)
- Decrease in Accounts payable (12,000)
- Decrease in Accruals (14,000)
Net Cash Flow from Operating Activities 62,100

Purchase of Gross fixed assets* (40,000)


Net Cash Flow from Investing Activities (40,000)

Increase in Notes payable 15,000


Change in LT Debt 6,000
Change in Common Stock 5,000
Increase in Capital surplus 1,000
- Payment of dividends** (33,200)
Net Cash Flow from Financing Activities (6,200)

Change in Cash 15,900

* Change in Net fixed assets + Depreciation


** Net income - (Change in Retained earnings)
Assignment 3.2:

Net income 1,800


+ Depreciation 1,600
- Increase in Accounts receivable (500)
- Increase in Inventories (300)
+ Increase in Accounts payable 1,100
+ Increase in Accruals 500
Net Cash Flow from Operating Activities 4,200

Purchase of Gross fixed assets* (2,400)


Net Cash Flow from Investing Activities (2,400)

Change in LT Debt -
Change in Common Stock -
- Payment of dividends** (1,200)
Net Cash Flow from Financing Activities (1,200)

Change in Cash 600

* Change in Net fixed assets + Depreciation


** Net income - (Change in Retained earnings)
Assignment 3.3:
Net income 32,000
+ Depreciation* 30,000
+ Decrease in Accounts receivable 7,000
- Increase in Inventories (30,900)
+ Increase in Accounts payable 8,000
- Decrease in Accruals (1,000)
Net Cash Flow from Operating Activities 45,100

Purchase of Gross fixed assets** (40,000)


Net Cash Flow from Investing Activities (40,000)

Change in Notes payable (3,000)


Change in LT Debt 15,000
Change in Common Stock 2,000
Change in Capital surplus 4,000
- Payment of dividends*** (24,900)
Net Cash Flow from Financing Activities (6,900)

Change in Cash (1,800)

* Change in Accumulated depreciation


** Change in Gross fixed assets
*** Net income - (Change in Retained earnings)
CHAPTER 3 – Answers to Additional Problems and Questions

Problems:
1. Raise prices, reduce growth rate, speed up collections, hold smaller inventory levels, etc.

2. A reduction or non-payment of a dividend, the substitution of a stock dividend for a cash


dividend, late payments to suppliers or other creditors, bounced payroll checks, etc.
3.
July Balance Sheet
Cash 15,000 Debt -
Accounts receivable 35,000 Stock 40,000
Inventories 28,000 Retained earnings 38,000
Total assets 78,000 Total claims 78,000

4. The growth in sales slowed to zero percent per year. Note that outflows for this company are
based on the current month’s sales level. That is, outflows in February are based on February
purchases (which are based on February sales). Outflows in March are based on March
purchases (which are based on March sales). And so on. Conversely, inflows are based on
sales that occurred one month ago. Thus, inflows in February are based on January sales and
inflows in March are based on February sales. In periods of rapid growth, this one-month lag
can cause outflows to exceed inflows even though revenue exceeds costs for that period.
When sales growth slows to zero, the lag effectively disappears since sales in each month are
the same. In this case, inflows minus outflows are essentially the same as revenue minus cost.

5. Depreciation expense (2004) = $2,000.


Depreciation expense (2005) = $4,000.
Depreciation expense (2006) = $2,000.

6. Change in gross fixed assets (2004) = $20,000.


Change in gross fixed assets (2005) = $40,000.
Change in gross fixed assets (2006) = $20,000.
7. Change in Net fixed assets + Depreciation (2004) = 18,000 + 2,000 = $20,000.
Change in Net fixed assets + Depreciation (2005) = 36,000 + 4,000 = $40,000.
Change in Net fixed assets + Depreciation (2006) = 18,000 + 2,000 = $20,000.
8.
Net income 26,000
+ Depreciation 2,000
- Increase in Accounts receivable (4,000)
- Increase in Inventories (2,000)
- Decrease in Accounts payable (2,000)
Net Cash Flow from Operating Activities 20,000

Purchase of Gross fixed assets (15,000)


Net Cash Flow from Investing Activities (15,000)

Change in Notes payable 3,000


Change in LT Debt 8,000
Change in Common Stock 15,000
- Payment of dividends (18,000)
Net Cash Flow from Financing Activities 8,000

Change in Cash 13,000


9. a. Total dividend paid = NI – change in R. E. = 3200 – (4000-2800) =$2000
b. Change in GFA/purchase = change in NFA + dep = (14,900-13,300)+1800 = $3400
GFA in 2006 = GFA of 2005 + purchase =20,000+ 3400 = $23, 400

10.
Net income 3,200
+ Depreciation 1,800
+ Decrease in Accounts receivable 500
- Increase in Inventories (600)
+ Increase in Accounts payable 1,800
- Decrease in Accruals (400)
Net Cash Flow from Operating Activities 6,300

Purchase of Gross fixed assets (3,400)


Net Cash Flow from Investing Activities (3,400)

Change in Notes payable 400


Change in LT Debt (1,300)
Change in Common Stock 600
- Payment of dividends (2,000)
Net Cash Flow from Financing Activities (2,300)

Change in Cash 600

6,300 – 3,400 – 2,300 = 600 (check)


11. Net cash flow from Operating Activities = $4,500
Net cash flow from Investing Activities = ($12,200)
Net cash flow from Financing Activities = $8,200

12. Net cash flow from Operating Activities = $470,000


Net cash flow from Investing Activities = ($350,000)*
Net cash flow from Financing Activities = ($110,000)
*Note that Long-term investments are an investing activity. An increase in Long-term
investments represents an outflow of cash.

13. a. $94,200 b. $113,500 c. $19,300


14. NCF Operating Activities = $22,900; NCF Investing Activities = ($23,200); NCF Financing
Activities = $100.
15. a. $1.96 million (compute net income and then add back depreciation, since depreciation is
not a cash flow).
b. $400,000 (note that when depreciation increases, taxes decrease. This is why depreciation,
although not a cash flow, can affect net cash flow from operations. Specifically, by
changing the amount of taxes a company owes.

16. a. ($6,000) - note that change in cash and marketable securities = ($4,000)
b. ($2,000) c. ($28,000) d. $26,000
The sum of these three cash flows = -$4000, which is the change in cash and marketable
securities. Marketable securities, as explained in chapter 2, are an alternative form of cash.
They are very short-term investments that generate a positive return; they can be converted
into cash very quickly. Essentially, cash that earns a return.
CHAPTER 4 – Solutions to Assignment Problems

Assignment 4.1:
1. a.
2005 2006
Net sales 100.0% 100.0%
Cost of goods sold 62.9% 64.1%
Gross profit 37.1% 35.9%
Operating expenses 27.1% 26.5%
Operating income 10.0% 9.5%
Income taxes 4.2% 4.1%
Net income 5.8% 5.4%

Note that the gross profit margin deteriorates by 1.2% and Operating profit margin deteriorates
by 0.5%. Company has been able to offset some of the gross margin deterioration by controlling
operating expenses – either real or manipulation of discretionary expenses.

2. (Note that some of the ratios you compute for this problem will look nonsensical. This is
done on purpose to stress computation of the ratios. Hopefully you will note that a company
with some of these ratios – in particular ACP and Inventory turnover, would probably be in
very poor shape).

2005 2006
Current ratio 2.3 2.4
Average collection period 288.2 365
Inventory turnover ratio .40 .39
Total asset turnover ratio .28 .24
Payables ratio 488.3 491.8
Debt ratio 57% 35%
Cash conversion cycle 715.5 805.1
Net profit margin 5.8% 5.4%
Return on equity 3.8% 3.2%
Assignment 4.2:
a. Total dividends = Net income – Change in Retained earnings = 1,600,000 – 600,000 =
1,000,000.
b. Number of shares = Common stock/ Par value per share = 1,500,000/.25 = 6,000,000 shares.
c. Number of shares = Common stock/ Par value per share = 1,900,000/.25 = 7,600,000 shares.
d. Dividends per share = Total dividends / Number of shares = 1,000,000/7,600,000 = 13 cents.
e. ROE = Net income / Total Equity = 1,400,000/ (1,500,000 + 1,000,000 + 1,000,000) = 40%.
f. ROE = Net income / Total Equity = 1,600,000/ (1,600,000 + 2,400,000 + 1,900,000) = 27%.
g. ROE (Dupont) = Net income/Sales x Sales/Total assets x Total assets/Total equity.
ROE (2002-Dupont) = .200 x 1.167 x 1.714 = .40 = 40%
ROE (2003-Dupont) = .178 x 0.900 x 1.695 = .27 = 27%
Net profit margin declined in 2003 by 11% (from 20% to 17.8%), Total asset turnover ratio
declined by 23% (from 1.714 to 0.900) and the equity multiplier declined by 1.1%. The
major reason for the deterioration in ROE is a decrease in the efficiency of asset utilization.
Secondary reason is a decline in profitability.

Assignment 4.3:

The best approach to do these “fill in the balance sheet” type problems is to use any given values
(in this case, Total assets = $1,000,000) to compute as many related values as possible.

For example, given that Assets (A) = 1,000,000, then from the fact that the Debt ratio (Total
liabilities/Total assets) = 60%, Total liabilities = 600,000. Since Total assets = Total liabilities +
Total equity (basic accounting relationship), Total equity must be 400,000.

On the given balance sheet, Total liabilities = Accounts payable + Long-term debt, and since
Long-term debt is given as 180,000, Accounts payable = 420,000. Also, on the given balance
sheet, since Total equity = Common stock + Retained earnings, and since Retained earnings is
given as 200,000, Common stock = 200,000.

Next use, Average collection period = 45 days = (Accounts receivable/Sales) x 360. From Total
asset turnover = Sales/Total assets = 2.0, Sales = 2,000,000. Thus,
45 = (Accounts receivable/2,000,000) x 360, so Accounts receivable = 250,000.

Then use, Inventory turnover ratio = 5 = COGS/Inventory. From Gross profit margin = 25% and
Sales = 2,000,000, COGS = 1,500,000. Thus,
5 = 1,500,000/Inventory, so Inventory = 300,000.

Then use Quick ratio = 1.0 = (Current assets - Inventory)/(Current liabilities). Note that Current
assets for this balance sheet = Cash + Accounts receivable + Inventory, so Current assets –
Inventory = Cash + Accounts receivable. Also note that for this balance sheet, Current liabilities
= Accounts payable. Thus,
1 = (Cash + 250,000)/(420,000), so Cash = 170,000.

Finally, Cash + Accounts receivable + Inventory + Fixed assets = Total assets. Thus, Fixed
assets = 280,000.

Summarizing:
Cash 170,000 Accounts payable 420,000
Accounts receivable 250,000 Current liabilities 420,000
Inventories 300,000 Long-term debt 180,000
Current assets 720,000 Total liabilities 600,000
Fixed assets 280,000 Common stock 200,000
Total assets 1,000,000 Retained earnings 200,000
Total claims 1,000,000
2. Sales = 2,000,000; COGS = 1,500,000
3. CCC = Inventory conversion period + Average collection period – Payables period
i. = 72 + 45 – 100.8 = 16.2 days.
4. ROE = Net profit margin x Total asset turnover ratio x (Total assets/Equity)
= .10 x 2.0 x (1,000,000/400,000) = .50 = 50%

CHAPTER 4 – Answers to Additional Problems and Questions

Problems:
1. A grocery store. Would you want to shop at a grocery store that had an inventory turnover
ratio of 2? That is, all items (fruit, meat, vegetables, etc.) turned over (i.e., sold) on average
only once every 6 months!
2. High inflation can distort the relationship between book values of assets and true economic
values. For example, if a firm has land on its balance sheet listed in value at $1,000 and
inflation in land values in running at 40 percent annually, 5 years later the market value of
the land would be worth approximately $5,400 but would only be listed on the balance sheet
as being worth $1,000. A similar distortion would apply to plant and equipment.
For short-term assets, however, values would, in general, adjust with inflation. If inventory
turns over relatively rapidly, each turnover would adjust inventory values upward. The same
would be true of accounts receivable. Thus, the relationship between the book value of short-
term assets and long term assets will be distorted. Finally, since the income statement is a
flow measure, high inflation usually affects all revenues and costs in a similar manner.
Assume that a firm has revenue of $1,000 and costs (total) of $800 and thus profit of $200. If
inflation is 50 percent and it impacts revenue and costs in a similar manner, then revenue will
grow to $1,500 (due solely to inflation) and costs will increase to $1,200 causing profit to
grow to $300. Thus inflation can cause a dramatic increase in net income (even more
pronounced if some costs are fixed). Now consider the impact of this increase on a ratio such
as ROA (recall from above that assets do not necessarily grow in line with inflation).
3. The current ratio is CA/CL. A declining ratio can either be due to a relative decline in CA or
a relative increase in CL. Since the inventory turnover ratio and the ACP are constant
through time, inventory and accounts receivable appear to be relatively stable. The increase
in the Payables period would indicate a relatively rapidly increasing Accounts payable value,
which would cause a relative increase in CL and therefore a decline in the current ratio. The
analyst should investigate the causes of the increase in AP perhaps a new supplier to the firm
offers more lax credit terms than the previous supplier.
4. 72%; 14.4%; 45%
5. CA = 2(CL) = 200,000. (CA - Inv)/100,000 = 1.5; thus, Inv = 50,000.
6. HINT: For problems where you are only given ratios and NO actual dollar amounts, assume
that assets are $100 and solve for all other numbers - then find the answer.
Thus, assume A = 100.
From ROA = NI/A; .20 = NI/100; thus, NI = 20.
Then, from ROE = NI/E; .50 = 20/E; thus, E = 40.
Then, from the definition, A = D + E; 100 = D + 40; thus, D = 60. (NOTE: this relationship
can be used for most of these types of problems).
Finally, compute the debt ratio = D/A = 60/100 = .60 = 60%.
7. Assume A = 100. Then from S/A = 4, S = 400. Then from D/A = .20, D = 20. Then from A =
D + E, E = 80. Then from NI/E = .10, NI = 8.
Finally, compute net profit margin = NI/S = 8/400 = 2%.
8. For this problem, you are given a dollar amount, so DO NOT assume that A = 100. Instead,
from ACP =(AR/S) x 360; 20 = (1,000/S) x 360; thus, S = 18,000. Then from NI/S = .05, NI
= 900. Then from S/A = 2, A = 9,000. Then from D/A = .75, D = 6,750. Then from A = D +
E, E = 2,250. Finally, ROE = NI/E = 900/2,250 = 40%.
9. Net profit margin = 8%; Debt ratio = 52%
10. ROA = 12.5%; ROE = 16.7%
11. Net profit margin = 8%; Debt ratio = 52%
We know that if we solve for x in (1950+x)/(800+x) = 2, we get the maximum amount by
which inventory = bank debt =x can increase without violating a current ratio of 2.Solving
for x we get, x = 350. In this case, Inv will be 1,050. Thus, (CA-Inv)/CL = (1950+350-1050)/
(800+350), and Quick ratio = 1.087.
12. Assume A = 100. Then D = 75 and E = 25 (recall A = D + E). Thus, Equity multiplier = A/E
= 100/25 = 4.
13. Answers assume a 360-day year.
a. $100
b. $450
c. $150
d. 5.7%
e. $400
f. $500

14. ROE = 42% (since all equity financed, equity = total assets)
15. Max Increase in debt = 1.4
16. B
17. From ACP = (ARx360)/S, 60 = (150,000x360)/S, S = 900,000. From NI/S = .04, NI =
36,000. From D/A = .64 and A = 3,000,000, D = 1,920,000. From A = D + E, E = 1,080,000.
Then, ROE = NI/E = .033 = 3.3%.
18. Note that CA = Cash & Mkt. Sec. + AR + Inv. and Inventory turnover here is defined as
S/Inv. From CA/CL = 3 and CA = 810,000, CL = 270,000. From (CA-Inv)/CL = 1.4 and
note above and Cash & Mkt. Sec. = 120,000, AR = 258,000. From note above, now Inv =
810,000 - 120,000 - 258,000 = 432,000. From S/Inv = 6, S = 2,592,000. From ACP =
(ARx360)/S, ACP = 35.8 days.
19. From NI/S = .06, NI = 120,000. From tax rate = .40 and from NI = EBT - (.40)x(EBT), EBT
= 200,000. To check, note that 200,000 x .4 = taxes of 80,000 and EBT - taxes = 120,000.
From EBT = EBIT - Interest, EBIT = 220,000. Thus, TIE = EBIT/Interest = 11 times.
20. Assume that currently A = 100. Then, currently, from debt ratio = .5, D = 50 and E = 50.
From S/A = .25, S = 25. From NI/S = .10, NI = 2.5. Using these numbers note that ROE =
5% (they want to double the current ROE from 5% to 10%) checks out. For new, assume that
S and A remain the same such that S/A will remain the same. Thus, S = 25 and A = 100.
From NI/S = .14, NI = 3.5 For ROE = NI/E to be .10, E must be 35. From A = D + E, D = 65
and thus, debt ratio = D/A = 65%.
21. Assume A = 100. From D/A = .35, D = 35 and E = 65. From NI/E = .15, NI = 9.75. From
S/A = 2.8, S = 280 Thus, NI/S = .0348 = 3.48%.
22. For these conversion type problems, note that if D = D/E and E = 1, D/E will be the stated
amount. Thus, for the first one, if D = 2.5 and E = 1, then D/E = 2.5. From A = D + E, A =
2.5 + 1 = 3.5. Then, D/A = 2.5/3.5 = .714 = 71.5% and A/E (the equity multiplier) = 3.5/1 =
3.5 times. For the second one, let D = 1 and E = 1 (thus, D/E = 1). Then A = D + E and A =
2. Thus, D/A = .5 and A/E = 2. For the third one, let D = .6 and E = 1. Then, A = 1.6. Thus,
D/A = .375 = 37.5% and A/E = 1.6 times.
23. From NI/S = .075, NI = 150,000.
From NI/E = .24, E = 625,000
From NI/A = .15, A = 1,000,000
Thus, Total liabilities = NP + AP + LT Debt = 375,000.
From S = 2,000,000 and Gross profit = 400,000, COGS = 1,600,000.
From COGS/Inv = 4, Inv = 400,000.
From (APx360)/COGS = 20, AP = 88,889.
From ACP = (ARx360)/S = 40, AR = 222,222.
From CA/CL = 3.8 and CA = AR + Inv = 622,222, CL = 163,743.
From CL = Notes payable + AP, Notes payable = 74,854.
Thus, long-term debt = 375,000 - 74,854 - 88,889 = 211,257.
Finally, A = 1,000,000 = AR + Inv + Fixed Assets.
Thus, Fixed assets = 377,778.
Summarizing, AR = 222,222; Inv = 400,000; Fixed assets = 377,778;
Total assets = 1,000,000; Notes payable = 74,854; AP = 88,889;
Long-term debt = 211, 257; Equity = 625,000;
Total liab & eq. = 1,000,000.
24. See if you can derive the following answer on your own. Hint, compute Accounts receivable
first, and then use the Quick ratio and Current ratio to compute Current liabilities and
Inventories.
Accounts receivable 400 Current liabilities 400
Inventories 800 Long-term debt 2000
Fixed assets 2800 Equity 1600
Total assets 4000 Total claims 4000
CHAPTER 5 – Solutions to Assignment Problems
Assignment 5.1:
Balance Sheet
FreshFish, Inc.
Years ending December 31, 2006 and 2007
2006 2007
Cash $ 20,000 $ 30,000
Accounts receivable 10,000 15,000
Inventory 80,000 120,000
Total Current Assets $110,000 165,000
Net plant and equipment $430,000 645,000
Total assets $540,000 $810,000

Notes payable 15,000 15,000


Accounts payable to suppliers 50,000 75,000
Accruals 5,000 7,500
Total current liabilities $ 70,000 97,500
Long-term debt 190,000 190,000
Common stock ($2.00 par value) 20,000 20,000
Capital surplus 150,000 150,000
Retained earnings 110,000 138,350
Total Liabilities and Equity $540,000 $595,850

2. OFN = $810,000 – $595,850 = $214,150


3. 540,000(.50) – 55,000(.50) – 900,000(1.50)(.035)(.60) = 270,000 – 27,500 – 28,350
= $214,150

Assignment 5.2:
You can answer this question for all parts using the OFN equation. Note that in that equation, the
last part is the change in retained earnings due to projected profit that is reinvested back into the
firm. This part of the equation is given as:
[(S0 + (g)(S0)](NPM) – Div0]
The first part of this equation is projected net income. With some minor mathematical
rearrangement, it can be written as (S0)(1+g)(NPM). The last term in the equation is the amount
of net income that is paid out in dividends. In the equation, it is given as a constant. For
Assignment 6.2, it is stated as a percent of net income (i.e., “Tabler pays out 40 percent of all its
annual profit in dividends”). If 40% of profit is paid out, then the remainder (i.e., 60%) must be
retained. Mathematically, this can be written as:
(S0)(1+g)(NPM)(.60)
Thus, the OFN equation becomes:
OFN = (TA0)(g) – [(L0)(g) + (S0)(1+g)(NPM)(.60)]
And filling in all other given values:
OFN = (1,220,000)(g) – [(500,000)(g) + (4,000,000)(1+g)(.05)(.60)]
= 1,220,000g – 500,000g – 120,000g – 120,000
= 600,000g – 120,000.
a. For g = 0% = 0, OFN = -120,000
b. For g = 10% = .10, OFN = -60,000
c. For g = 20% = .10, OFN = 0
d. For g = 30% = .10, OFN = 60,000
e. For g = 40% = .10, OFN = 120,000
f. For g = 50% = .10, OFN = 180,000

2. OFN = 0 when g = 20% (see c above).

Assignment 5.3:
a. Earnings per share = Net income divided by number of shares of common stock
outstanding. Net income = (.05)(90,000) = 4,500. Number of shares = Common stock
value of balance sheet divided by par value per share = 60,000/2 = 30,000. Thus,
Earnings per share = 4,500/30,000 = .15 = 15%

b. Dividends = Net income – Change in retained earnings = 4,500 – (30,000 – 28,200) =


2,700. Dividend payout ratio = Dividends divided by Net income (that is, the ratio
measures the percent of net income that is paid out in dividends).
Dividend payout ratio = 2,700/4,500 = .60 = 60%.

c. Use the equation: OFN = (TA0)(g) – [(L0)(g) + [(S0 + (g)(S0)](NPM) – Div0].


OFN = (200,000)(.40) – [(60,000)(.40) + [(90,000 + (.40)(90,000)](.05) – 4,000]
= 80,000 – 24,000 – 2,300
= 53,700.
2. Construct the proforma income statement:
Sales $330,000
Operating Costs (70%) 231,000
Fixed costs 60,000
EBIT 39,000
Interest 10,000
EBT 29,000
Taxes (40%) 11,600
Net income 17,400
Dividends 9,000
Addition to Retained earnings 8,400

Now, construct the proforma balance sheet:


Current assets 110,000 Notes payable 20,000
Net fixed assets 55,000 Accounts payable 22,000
Total assets 165,000 Accruals 11,000
Long-term debt 50,000
Equity 58,400
Total claims 161,400
OFN = 165,000 – 161,400 = 3,600.
CHAPTER 5 – Answers to Additional Problems and Questions

Problems:
1. Reduce the dividend payout rate or increase the net profit margin.
2. a. and b.
Apr May Jun Jul Aug Sep
Cash Collections 75,000 100,000 300,000 500,000 900,000 500,000
Cash Expenses 350,000 350,000 350,000 350,000 350,000 350,000
Net Cash Flow (NCF) (275,000) (250,000) (50,000) 150,000 550,000 150,000

Begin. Cash + NCF 75,000 (175,000) - 150,000 600,000 625,000

Ending ST Loan Balance - 225,000 275,000 125,000 - -


Ending Cash Balance 75,000 50,000 50,000 50,000 475,000 625,000

NOTE: If ending cash balance is greater than 50,000, the ST loan is zero. If ending cash balance is less than
50,000, the ST loan will grow so as to make ending cash equal to 50,000.

c. Accounts receivable balance (Apr) = 100,000


Accounts receivable balance (May) = 300,000
Accounts receivable balance (Jun) = 500,000
Accounts receivable balance (Jul) = 900,000
Accounts receivable balance (Aug) = 500,000
Accounts receivable balance (Sep) = 200,000

3. Annual Inventory Related Interest Cost (current) = 25 x 500 x .10 = $1,250


Annual Inventory Related Interest Cost (proposed) = 15 x 500 x .10 = $750
So, reduction in annual inventory related interest cost = $750.
Reduction in Rent = $5,000 annual savings.
So, total benefit of proposed change = $5,750

Number of Inventory orders per year (current) = 12


Number of Inventory orders per year (proposed) = 36
Increase in Inventory order cost = (36)($250) = $6,000

Thus, benefit – cost = $5,750 - $6,000 = ($250) – DO NOT CHANGE POLICY.


4.
COFFYS
Proforma Balance Sheet
For the Year Ending June 30, 2004

Cash $14,000 Notes payable $10,000


Accounts receivable 11,200 Accounts payable 35,000
Inventories 100,800 Accruals 7,000
Current Assets 126,000 Current Liabilities 52,000

Gross fixed assets 1,148,000 Long term bank loan 400,000


Accumulated dep. (84,000) Common stock 200,000
Net fixed assets 1,064,000 Retained earnings 226,000
Total assets 1,190,000 Total Liab. & equity
878,000
All asset accounts grow by 40% and accounts payable and accruals (i.e., spontaneous liabilities)
grow by 40%. Financing variable (i.e., Notes payable, Long-term bank loan and Common stock)
values stay fixed. The change in Retained earnings is due to net income that is not paid in
dividends. Specifically,
Change in RE = (2003 Sales)(1 + Projected growth rate)(Net profit margin) – Dividends
= (800,000)(1.4)(.05) – 40,000
= 16,000
And thus proforma Retained earnings = 210,000 + 16,000 = 226,000.
Outside funds needed = 1,190,000 – 878,000 = 312,000.

Check:
OFN = (TA0)(g) – [(L0)(g) + [(S0 + (g)(S0)](NPM) – Div0]
= (850,000)(.40) – [(30,000)(.40) + [(800,000 + (.40)(800,000)](.05) – 40,000
= 340,000 – [12,000 + 16,000]
= 312,000.

5.
COFFYS
Proforma Balance Sheet
For the Year Ending June 30, 2004

Cash $14,000 Notes payable $ 8,400


Accounts receivable 11,200 Accounts payable 35,000
Inventories 100,800 Accruals 7,000
Current Assets 126,000 Current Liabilities 50,400

Gross fixed assets 1,148,000 Long term bank loan 400,000


Accumulated dep. (84,000) Common stock 200,000
Net fixed assets 1,064,000 Retained earnings 226,000
Total assets 1,190,000 Total Liab. & equity 876,400
All asset accounts grow by 40% and accounts payable and accruals (i.e., spontaneous liabilities)
grow by 40%.

To solve for Notes payable, use the fact that the current ratio will be 2.5. Since you have
projected current assets = 126,000, this implies that projected current liabilities will be 50,400.
With Accounts payable = 35,000 and Accruals = 7,000 (due to spontaneous liabilities grow by
40%), Notes payable = 52,000 – 35,000 – 7,000 = 8,400.

The remaining financing variable (i.e., Long-term bank loan and Common stock) values stay
fixed. The change in Retained earnings is due to net income that is not paid in dividends.
Specifically,
Change in RE = (2003 Sales)(1 + Projected growth rate)(Net profit margin) – Dividends
= (800,000)(1.4)(.05) – 40,000
= 16,000
And thus proforma Retained earnings = 210,000 + 16,000 = 226,000.

Long term outside funds needed = 1,190,000 – 876,400 = 313,600. (This money will come from
some combination of additional long-term debt and additional common stock).

6.
COFFYS
Proforma Balance Sheet
For the Year Ending June 30, 2004

Cash $14,000 Notes payable $21,000


Accounts receivable 11,200 Accounts payable 35,000
Inventories 100,800 Accruals 7,000
Current Assets 126,000 Current Liabilities 63,000

Gross fixed assets 1,148,000 Long term bank loan 413,000


Accumulated dep. (84,000) Common stock 200,000
Net fixed assets 1,064,000 Retained earnings 226,000
Total assets 1,190,000 Total Liab. & equity
902,000
All asset accounts grow by 40% and accounts payable and accruals (i.e., spontaneous liabilities)
grow by 40%.

To solve for Notes payable, use the fact that the current ratio will be 2.0. Since you have
projected current assets = 126,000, this implies that projected current liabilities will be 63,000.
With Accounts payable = 35,000 and Accruals = 7,000 (due to spontaneous liabilities grow by
40%), Notes payable = 63,000 – 35,000 – 7,000 = 21,000.

To solve for Long-term bank loan, use the fact that the debt ratio will be 40 percent. The debt
ratio = total liabilities divided by total assets. With total assets projected to be 1,190,000, total
liabilities will be (.40)(1,190,000) = 476,000. Since Current liabilities = 63,000, Long-term bank
loan must be 476,000 – 63,000 = 413,000.
The remaining financing variable (i.e., Common stock) value stay fixed. The change in Retained
earnings is due to net income that is not paid in dividends. Specifically,
Change in RE = (2003 Sales)(1 + Projected growth rate)(Net profit margin) – Dividends
= (800,000)(1.4)(.05) – 40,000
= 16,000
And thus proforma Retained earnings = 210,000 + 16,000 = 226,000.

Money needed from selling additional common stock = 1,190,000 – 902,000 = 288,000.

7. a. $101,000
b. - $24,000
(TA = 1250000*1.15 =1437,500, 60% of which is debt, Total equity needed = 0.4 of
1437,500 = 575,000. Now addition to R. E = 49,000. Total R. E = 300,000+49,000 =
349,000. Total equity fund available = (50,000+200,000+349,000) = 599,000. Therefore,
common stock needed (575,000-599,000) = -24,000
8. $130,400
9. $112,500
10. a. $150,000 b. $137,400 c. $124,800
11.
(a) 2003 Actual 2004 Proforma
Cash 400,000 500,000
Acc. receivable 900,000 1,125,000
Inventory 1,200,000 1,500,000
Net Prop. & Plant 2,500,000 3,125,000
Total Assets 5,000,000 6,250,000

Acc. payable 800,000 1,000,000


Long term debt 1,500,000 1,500,000
Common Stock 1,800,000 1,800,000
Retain earnings 900,000 1,140,000
Tot. Liab. & Equity 5,000,000 5,440,000

Total Outside Funds Needed = 810,000

(b) 2003 Actual 2004 Proforma


Cash 400,000 500,000
Acc. receivable 900,000 1,125,000
Inventory 1,200,000 1,500,000
Net Prop. & Plant 2,500,000 3,125,000
Total Assets 5,000,000 6,250,000

Acc. payable 800,000 1,000,000


Long term debt 1,500,000 2,750,000
Common Stock 1,800,000 1,800,000
Retain. earnings 900,000 1,140,000
Tot. Liab. and Equity 5,000,000 6,690,000
Additional Common Stock Needed =(440,000)

12. a. Total Outside Funds Needed = 2,235,000


b. Total Outside Funds Needed = 885,000
CHAPTER 6 – Solutions to Assignment Problems

Assignment 6.1:
1. FV = PV (1+r). Let FV = 2 and PV =1. Thus, 2 = 1(1+r). When you solve for r, you get r=1.
Expressed as a percent, the interest rate is 100%.
2. FV = PV (1+r); 7397 = PV(1+.0439) or PV = 7297/1.0439; Solve PV = 7,085.93.
3. FV = PV (1+r); 13000 = PV(1.08) or PV = 13000/1.08; Solve for PV = 12,037.04.
4. Note that you will only need 25000 – 4500 = 20,500 one year from today. Thus, FV = PV
(1+r); 20500 = PV(1.075) or PV = 20500/1.075; Solve for PV = 19,069.77
5. FV = PV (1+r); 14739 = PV(1.12) or PV = 14739/1.12; Solve for PV = 13,159.82.

Assignment 6.2:
1. FVn = PV (1+r)n. Let FV = 2 and PV =1. Thus, 2 = 1(1+r)2 or 2½ = (1+r). When you solve for
r, you get r=.4142. Expressed as a percent, the interest rate is 41.42%.
2. PV = FV1(1/1+r)1 + FV2(1/1+r)2 ; PV = 3200(1/1.06)1 + 7300(1/1.06)2 ; PV = 3018.87 +
6496.97 = 9,515.84.
3. Note that the first deposit will grow for one year – that is, it will grow to become 7448 (1.07)
= 7969.36. When you add the extra 2476, you will have a total of 10,445.36 in your account.
4. Continuing from #3, the 10,445.36 will grow again by 7 percent to be 10445.367(1.07) =
11,176.54.
5. 15000 = X(1.12)2 + X(1.12)1 = X [(1.12)2 + (1.12)1] = X[2.3744]. Thus, X = 6,317.39.

Assignment 6.3:
1. FV2 = Deposit0 (1+r)2 + Deposit1 (1+r)1. (Note that the deposit made today (at t=0) will earn
interest for 2 years and the deposit made one year from today will earn interest for 1 year).
Thus, 4000 = 8000(1.04)2 + Deposit (1.04)1; 4000 = 8652 + Deposit (1.04). Solve for Deposit
and you get Deposit = -4,473.85. The negative sign implies that you will withdraw this
amount at the end of year one.
2. The question is what is the present value of the investment. That is, what is the present value
of 6500 one year from today + 5000 two years from today. We then compare the value of the
investment with the cost. If value > cost, you should buy. If value < cost, you should not buy.
PV = FV1(1/1+r)1 + FV2(1/1+r)2 ; PV = 6500(1/1.12)1 + 5000(1/1.12)2 ; PV = 5803.57 +
3958.97 = 9,789.54. Do not make investment.
3. 538 = 500 (1+r)1; 1.0760 = 1+r; r = .0760 = 7.6%
4. (1 + rnominal) = (1 + rreal) x (1 + i); (1 + rnominal) = (1.08) x (1.04) = 1.1232. rnominal = .1232 =
12.32%.
5. (1 + rnominal) = (1 + rreal) x (1 + i); (1.122) = (1 + rreal) x (1.036); (1 + rreal) = 1.083. rreal = .083 =
8.3%.
6. (1 + rnominal) = (1 + rreal) x (1 + i); (1.086) = (1.047)(1 + i); (1 + i) = 1.0372. i = .0372 = 3.72%
CHAPTER 6 – Answers to Additional Problems and Questions

Problems:
1. 10.24%
2. 8.112%
3. 5.77%
4. 5.15%
5. 1.92%
6. 10.04%
7. 1294.80
8. 4108.90
9. 237.60
10. 4917.12
11. 969.31
12. 1696.25
13. 3752.35
14. 612.83
15. E(R) [i.e., Expected Return] on first = 11.5%;E(R) on second = 12.1%
Choose both since Expected Return > Required Return. NOTE: This problem implicitly
assumes that these two investments are of equal risk. Unless otherwise explicitly stated, for
all problems in this book, we will assume that all comparable investments are of equal risk.
16. E(R) on first = 10.2%; E(R) on second = 7.48%
Choose neither since for both E(R) < Required Return.
17. (d) is correct. For (a), PV must be less than 432 since interest rate is greater than 0.for (b), the
FV (t=1) must be greater than the PV (t=0) value since interest rate is greater than 0.for (c),
you would never pay more than the simple sum of all future cash flows (i.e., 2,300+2,300 =
4,600 < 6,600)
18. 70,661.16
19. 1,680.00; 1,915.20
20. In the following equation, solve for X (note that ^2 means raised to the second power, or int
this case, squared): 10,000 = 7,000 (1/1.13)^1 + X(1/1.13)^2 ==> X = 4,859
21. In the following equation, solve for r: 4,100 = 3,500 (1+r)^2 ==> r = 8.233%
22. Buy the two year subscription because the PV of buying a one year subscription today and
another one year subscription one year from today = 48 + 48(1/1.10) = 91.64 which is greater
than the two year subscription price of 70. This answer of course assumes that you actually
want to read this magazine for two years!
23. Assume that the two payments are due one month from today and 2 months from today.
Payoff = 791.09
24. Compare 20,000 - 2,850 = 17,150 to the PV of 20,000 two years from today.
20,000(1/1.08)^2 = 17,146.78. Since 17,150 > 17,146.78, choose (b) because it is "cheaper."
CHAPTER 7 – Solutions to Assignment Problems

Assignment 7.1:
1. PV = -30000, FV = 49000, n = 5, Compute I/Y = 10.31%.
PV = -73000, FV = 128000, n = 7, Compute I/Y = 8.35%.
2. Use cash flow register: CF0 = 0, C01 = 22000, F01 = 1, C02 = 27500, F02 = 1, C03 =
33000, F03 = 1, C04 = 35000, F04 = 1; Compute NPV (with I = 6%) = 100,660.33
3. Use cash flow register to find NPV. Then find FV of this amount. Thus, CF0 = 11000, C01 =
13000, F01 = 1, C02 = 17400, F02 = 1, C03 = 12800, F03 = 1, C04 = 9600, F04 = 1; C05 =
17200, F05 = 1; Compute NPV (with I = 8%) = 66878.10. Now find the future value of this
amount 10 years from today. PV = 66878.10, I = 8, n = 10, Compute FV = 144,384.81.

Assignment 7.2:
1. PV = -28000, fv = 30000, I = 6, n = 10, Compute PMT = 1,528.26.
2. First find F of deposits. PMT = 2500, I = 8, n = 20, Compute FV20 = 114404.91. Now find
withdrawals. PV = 114404.91, n = 25, I = 8, Compute PMT = 10,717.31
3. First find NPV of all cash flows with the unknown cash flow assumed to be 0. That is, CF0 =
5000, C01 = 0, F01 = 10, C02 = -60000, F02 = 1, C03 = 0, F03 = 2, C04 = 25000, F04 = 1,
C05 = 0, F05 = 11, C06 = -1500000, F06 = 25; Compute NPV (with I = 7%) = 335884.60.
Now find PMT. PV = 335884.60, I = 7, n = 25, Compute PMT = 28,822.43.
4. First find PV20 of perpetuity = 30000/.12 = 250000 (note this is value in year 20).
CF0 = 0, C01 = 12000, F01 = 3, C02 = 17000, F02 = 4, C03 = 21000, F03 = 8, C04 = 24000,

F04 = 4; C05 = 274000, F05 = 1; Compute NPV (with I = 12%) = 154,486.56.

Assignment 7.3:
1. PV = -1, FV = 2, I = 7/2 = 3.5, Compute n = 20.1488 semi-annual periods = 10.07 years.
2. Bank A: 10.00%; Bank B: 10.04%; Bank C: 9.95%; Bank D: 9.93%; Bank E: 9.86%.
3. PMT = 140000, n = 10, I = 9, Compute PV = 898472.08. This is value need at the end of
year 20. Since first deposit will be made today, set calculator in BEGIN mode. Now, FV =
898472.08, I = 9, n = 20, Compute PMT = 16,111.89.
4. Annual payment = $8,652.62.

Beginning Ending
Year Balance Interest Payment Balance

1 40,000.00 3,200.00 $8,652.62 34,547.38


2 34,547.38 2,763.79 $8,652.62 28,658.56
3 28,658.56 2,292.68 $8,652.62 22,298.63
4 22,298.63 1,783.89 $8,652.62 15,429.90
5 15,429.90 1,234.39 $8,652.62 8,011.68
6 8,011.68 640.93 $8,652.62 0.00
CHAPTER 7 – Answers to Additional Problems and Questions

Problems:
1. a – more compounding periods per year creates a larger effective interest rate.
2. e
3. 17 years
4. 4.45%
5. $10,962.37
6. d
7. a
8. 19.56%
9. $704
10. e
11. e

12. This question is misleading. This is not an annuity problem.


For example, n=10, PMT=0, r = 10, FV = 5,000, PV = ?1,927.72
n=10, PMT=0, r= 10, PV = 5,000, FV = ? 12, 968.71

Rate PV FV
0 5,000.00 5,000.00
5 3,069.57 8,144.47
10 1,927.72 12,968.71
15 1,235.92 20,227.79
20 807.53 30,958.68
25 536.87 46,566.13
30 362.69 68,929.25
35 248.68 100,532.78
40 172.86 144,627.33
45 121.70 205,423.45
50 86.71 288,325.20
Plot of Present Values

6,000.00

5,000.00
Present Value

4,000.00

3,000.00 Series1

2,000.00

1,000.00

-
0 5 10 15 20 25 30 35 40 45 50
Interest Rate

Plot of Future Values

350,000.00
300,000.00
250,000.00
Future Value

200,000.00
Series1
150,000.00
100,000.00
50,000.00
-
0 5 10 15 20 25 30 35 40 45 50
Interest Rate

13. $15,129.38
14. $45,349.14
15. $26.97
16. $889.23
17. 2.74%
18. 4.47%
19. $96,969.53
20. 7.10%; 7.23%
21. $31,265.66
22. 8.654%
23. $7,669.12; $18,419.93
24. $76,175.84
25. $6,714.27
Alternative 1:
I do not think solution manual gave right answer, I did in two different method and this is what I
get
CF1 = 20,000, CF1 = 0, F1= 10 (assume that t = 12 means ending of year 11)
CF2 = -25,000, F2 = 4, (ending at t = 14 or beginning at t = 15)
Cf3 = 0, f3 = 2 (assume that t = 18 means ending of year 17)
Cf4= -33,000, f4 = 4
I = 8%, NPV = - 50,257.52 (this is the spending at t =0)
You have to save for this for 10 yrs
Pv = - 70,257.52, FV = 0, N = 10, 1/Y = 8, PMT = ? 7,489.85
Alternative 2:
Pv1) PMT= 33,000; n=4; FV=0; PV=-118, 044.2006; 1/y= 8, (Solved as ANN Due) at the end of
t =17
Pv2) PMT=0; n=7; 1/y=8; FV= 118, 044.2006, PV=- 68,877.65 at the end of t = 10
PV3) PMT= 25000; n=4; i/y= 8; FV=0; Pv= -89,427.42468 (Solved as ANN Due) at the end of t
=11
PV4) Fv= 89,427.42468; n=1; 1/y= 8, PMT=0, PV= -82,803.171 at the end of t = 10
Pt 5) Pv= -20,000, n=10, FV= (68,877.65 +82,803.171), 1/y=8, PMT= -7,489.85

26. $4,520,178.42 today; $2,883,820.96 (5,082,277.89 is correct answer) in five years.


27. $263.80
28. $59,739.98
29. $9.50
30. $482.09
When PMT = 0, it does not matter if you use END or BGN mode. But I suggest you always keep
your calculator at end mode, just to avoid problems.
Cf0 = 0, cf1 = 0, f1 = 59 (beg of t = 60 implies end of t = 59)
Cf2 = -2500, f2=16, I = 1.25, NPV = - 17,322.37
PV = - 17,322.37, N = 48, 1/y = 1.25, FV = 0, PMT = ? 482.09

31. Amount of payment that goes to principal = 7757.16; Amount that goes to interest = 1,698.84
32. $925,764
33. $86,303.09
34. $17,954.13
Correct method for the solution in END mode is as follows:
PV= -3000, PMT= -3000, n=4, I/y=9, FV= 17, 954. 13

35. $276.21
36. $167,790.24
37. $5,468.21
38. $61,534.10
39. $165,918.32
40. $71,474.07
41. $871.47
As a first step, calculate the PV of the cash flows (at 10.5%): cf0 = 0, Cf1=1700, f1 = 1,
Cf2=1800, f2 = 1, cf3 = 0, f3 =1, cf4 = 2000, f4 = 1, NPV =4354. 1026
Therefore, you are 5000-4354. 1026=645.897 “short” of the 5000. This represents the PV of the
missing cash flow. To calculate the actual cash flow, calculate the FV of the 645.897:
PV=645.897, N=3; I=10.5; PMT = 0; FV=??=871.47
To verify your answer, enter the 871.47 as Cf3 and calculate the PV (it works out
to 5,000)
CHAPTER 9

ASSIGNMENT 9.1

1.
a. Treat as a perpetuity. P = Coupon/Interest Rate = 82.50/0.08 = $1,031.25

b. P = 1000/(1.0815) = $315.24

c. N = 20; PMT = 70; I/Y = 8; FV = 1000; CPT PV. P = $901.82

2.
a. N = 60; FV = 1000; PMT = 35; I/Y = 4.5; CPT PV. P = $793.62

b. N = 80; FV = 1000; PMT = 25; I/Y = 2.25; CPT PV. P = $1,092.37

3. PV = -978; FV = 1000; PMT = 90; N = 14; CPT I/Y. Cost of Debt = YTM = 9.29%

4. N = 68; (i.e. 17x4 quarters) PV = -1020; FV = 1000; PMT = 20; CPT I/Y;
I/Y = 1.9467; Thus, Cost of Debt = YTM = 1.9467 x 4 = 7.79%

ASSIGNMENT 9.2

1. Preferred should be treated as a perpetuity as it pays a perpetual stream of preferred dividends.


Price = 2.25/0.11 = $20.45

QWE’s estimate of the required rate of return is higher than the actual required rate of return
investors are using to value the preferred stock.
The actual rate = 2.25/24 = 9.375%

2. Since the common stock pays constant dividends forever, the present value of the dividends is
obtained using the perpetuity formula, as in 1 above.

P = 1.50/0.09 = $16.67

3. P0 = D1 / (k-g) = 2.50/(0.10-0.03) = $35.71

4. First find P5 = D6 / (k-g) = $1(1.05)/(0.12-0.05) = 1.05/0.07 = $15.

Now P0 = 1/1.12 + 1/1.122 + 1/1.123 + 1/1.124 + (1+15)/1.125 = $12.12.


Alternative Solution: After finding P5 note that the cash flow lends itself to the use of the third
row keys on your calculator. Thus we can use our usual setup to find the present value.
PMT = 1; FV = 15; N = 5; I/Y = 12; CPT PV. P = $12.12
ASSIGNMENT 9.3
1.
P0 = D1/(k-g) = D0(1+g)/(k-g). We can make k the subject of this equation.

k = g + D0(1+g)/P0 = 0.04 + 2(1.04)/38 = 0.04 + 0.0547 = 9.47%

2. First determine D4 = D3(1+g) = 3(1.06) = $3.18

Then P3 = D4/(k-g) = 3.18/(0.14-0.06) = $39.75

Now, P0 = 2/1.14 + 1/1.142 + (3+39.75)/1.143 = $31.38

You can also use the CF (cash flow) procedure to do the calculations in the last step:

C01 = 2; F01 =1; C02 = 1; F02 =1; C03 = 42.75 (3+39.75); F03 = 1; I=14.

3.

D1 = 1.75(1.5) = 2.625; D2 = 1.7591.5)2 = 3.9375; D3 =1.75(1.52)(1.3) = 5.11875

D4 = 1.75(1.52)(1.32) = 6.654375; D5 = 1.75(1.52)(1.32)(1.2) = 7.98525;

D6 = 1.75(1.52)(1.32)(1.2)(1.07) = 8.5442175.

P5 = D6/(k-g) = 8.5442175/(0.15-0.07) = $106.80

Then P0 = 2.625/1.15 + 3.9375/1.152 + 5.11875/1.153 + 6.654375/1.154 +


(7.98525+106.8027)/1.155 = $69.50.

Again, you can use the CF procedure (probably easier) for the last step.

4. D1 = $1(1.35) = 1.35; D2 = 1(1.352) = 1.8225; D3 = 1(1.353) = 2.460375;

D4 = 2.460375(1.2) = 2.95245; D5 = 2.95245(1.2) = 3.54294;

D6 = 3.54294(1.2) = 4.251528; D7 = D8 = D9 = …… = D∞ = 4.251528.

After this, it is probably best to use the cash flow procedure for the last step:

C01 = 1.35; F01 = 1; C02 = 1.8225; F02 = 1; C03 = 2.460375; F03 = 1;

C04 = 2.95245; F04 =1; C05 = 3.54294; F05 = 1; C06 = 4.251528; F06 = 999.

Use I = 15. Then the (NPV) Price = $21.71


Alternatively, the last step can be done using the analytical method:

First find P6 = D7/k = 4.251528/0.15 = $28.34.

P0 = 1.35/1.15 + 1.8225/1.152 + 2.460375/1.153 + 2.95245/1.154 + 3.54294/1.155 +


(4.251528+28.34)/1.156 = $21.71

ADDITIONAL PROBLEMS

1. PV = -1100; N = 25; PMT = 90; FV =1000; CPT I/Y.

Market interest rate = YTM = 8.06%

2. To be indifferent between calling the bond and not calling the bond, the call price of $1,075
must be equal to the present value of the remaining payments. Thus:

PV = -1075; PMT = 80; N = 25; FV = 1000; CPT 1/Y. 7.34%

3. Use the usual procedure for finding the price of a bond. Just be careful to use the appropriate
YTM for I/Y for the various times.

a. $1,000 b. $1,087.45; c. $873.08; d. $974.69; e. $972.97

4. FV = 1000; PV = -1092; PMT = 50; N=40; CPT I/Y. I/Y = 4.5%. Thus YTM = 9%

5. FV=1000; N=8; PMT = 0; I/Y = 4. CPT PV. Price = $730.69

6. FV=1000; N=34; PMT = 40; I/Y = 3.625; CPT PV. P = $1,072.62

7. FV = 1000; N = 30; PMT = 35; PV = -825; CPT I/Y. I/Y = 4.585. YTM = 9.17%

8. YTM = 9.09%

9. P = $859.16

10. First find the yield to maturity of the zero coupon bonds.
N = 30; FV =1000; PV = -99.38; PMT = 0; CPT I/Y. YTM = 8%.
Now, the yield to maturity of the coupon bond is also 8%. Thus for the coupon bond:

N = 60; FV = 1000; PV = -886.88; I/Y = 4; CPT PMT. PMT = 35. Coupon Rate = 7%.

11. Price per share = 20000000/1000000 = $2. D = P x k = 2x0.11 = $2.20 per share.

12. P0 = D0(1+g)/(k-g) = 1.50(1.04)/(0.12-0.04) = 1.56/0.08 = $19.50

13. P0 = D0(1+g)/(k-g) = 4(1-0.05)/(0.20--0.05) = 4(0.95)/0.25 = $15.20


14. P0 = D0(1+g)/(k-g); 15 = 1(1+g)/(0.12-g); 1.80 -15g = 1+g; g = 0.8/16 = 5%

15. P0 = D0(1+g)/(k-g); 72.25 = 5.12(1.06)/(k-0.06); k = 13.51%

16. The first dividend that begins the constant growth forever is D5. Thus, we can find P4 using:

P4 = D4 (1+g)/(k-g) = 3.5(1.08)/(0.14-0.08) = $63.

Now, either use he cash flow procedure or the analytical method to solve for P0.

P0 = 2/1.14 + 1.50/1.142 + 2.50/1.143 + (3.50+63)/1.144 = $43.97

17. First let’s find the discount rate, k. 24 = 3/k. Thus k = 12.5%.

Now, with the 3 percent expected growth, the dividend one year from now, D1 = 3(1.03) = $3.09.
Therefore, P0 = 3.09/(0.125-0.03) = $32.53.

18. g = 2.14/2 – 1 = 7%. P0 = 2.14/(0.26-0.07) = $11.26

Take note of the fact that the question gave some pieces of information that were irrelevant to
solving the problem.

19. First determine k, the investors required rate of return. k = 2.10/15 = 14%.

Then P0 = 2.10(1.04)/(0.14-0.04) = $21.84.

20. D1 = 4(0.75) = 3; D2 3(0.86) = 2.58; D3 = 2.58(0.94) = 2.4252.

P3 = 2.4252(1.044)/(0.184-0.044) = 18.09. P0 = 3/1.184 + 2.58/1.1842 +


(2.4252+18.09)/1.1843 = 2.534 +1.8404 + 12.36 =16.73
CHAPTER 10

ASSIGNMENT 10.1
1.

Payback Discounted
NPV IRR PI Period Payback Per.

A $3,456.40 22.58% 1.38 2.8 years 3.22 years

B $3,885.38 20.66 1.35 3.09 3.59

C $5,583.82 19.48 1.24 3.11 3.44


Never Pays
D -$2,536.31 2.62 0.80 4.84 Back (∞)

2.
PMT = $26,000,000; FV = 20,000,000; N = 10; I/Y = 12; CPT PV.
PV (Benefits) = $153,345,263.50.
Thus, NPV = PV (Benefits) – PV (Costs) = 153,345,263.50 – 200,000,000 = - $ 46,654,736.50
IRR = 6.25%
PI = PV (Benefits)/PV (Costs) = 153,345,263.5/200,000,000 = 0.77

3. P0 = 10,000,000/(0.14-0.05) = $111,111,111.11

NPV = 111,111,111.11 – 100,000,000 = $11,111,111.11

The IRR is the discount rate that will make the present value of the benefits equal to the initial
cost (that is the discount rate that will make the NPV equal to zero). 100,000,000 =
10,000,000/(k-0.05);
k = IRR = 15%

PI = 111,111,111.11/100,000,000 = 1.11
ASSIGNMENT 10.2
1.
R NPV
0 -300
10 86.78
20 250
30 275.15
40 214.29
50 100
60 -46.86
70 -212.80
80 -388.89
2.

NPV Profile for Project A


400
300
200
100
0
NPV

-100 0 10 20 30 40 50 60 70 80 90
-200
-300
-400
-500
Discount Rate

The IRRs estimated from the graph are 7% and 57%

ASSIGNMENT 10.3

1. The net cash flow per month is 12x600 – 1800 = $5400.


Thus, PMT = $5400; FV = $500,000; N =120; I/Y = 1; CPT PV.
PV = $527,880.21

2. The net cash flow per year = 20x6000 – 33000 = $87,000.


Thus, PMT = 87000; N = 12; FV = 400000; I/Y = 12; CPT PV.
PV = $641,580.59. The offer of $675,000 exceeds the present value of cash flows from
the property. Therefore SELL.

3. YTM = 8.5% = Pretax cost of debt. After tax cost of debt = (1-0.3)x(8.5%) = 5.95%.
For common stock: 20=1.5(1.05)/(k-0.05). Solve this equation for k, the
investors required rate of return.

k = 0.05 + 0.07875 = 12.875%

Cost of capital = 0.25(5.95) + 0.75(12.875) = 11.14%


4. Bond. FV = 1000; N =50; PMT = 27.5; PV = -695; CPT I/Y. I/Y = 4.2249%. Since
coupons are paid semiannually, the YTM = 2x4.2249 = 8.45%
After tax cost of debt = (1-0.3)(8.45) = 5.915%

Stock. 26.5 = 2.1(1.055)/(k-0.055). Solving for k, we have: k= 13.86%

Cost of capital = 0.6(5.915) + 0.4(13.86) = 9.09%

Additional Questions and Problems

1.

PROJECT NPV IRR PI Payback Period Disc. Payback Period

A $603.58 31.79% 1.60 2.5 yrs 3.07 yrs

B -$456.15 7.93% 0.92 4 yrs Infinity

C -$3974.15 6.54% 0.86 3.67 yrs Infinity

D $3807.59 16.43% 1.22 3.70 yrs 4.22 yrs

2. The project has 2 IRRs because the cash flows change signs two times. The IRRs are 48.18%
and -50.55%. To solve for the IRR, you need to solve the following equation for r.
1996/(1+r) – 740/(1+r)2 -1010 = 0. You can use trial and error or you can solve using the
quadratic formula. For the quadratic, first multiply the equation through by (1+r)2 and simplify to
obtain a quadratic equation. Then use the quadratic formula.

If the cost of capital is 20%, the project should be accepted because it has a positive NPV. NPV
= $139.44.

3. a. $100,000
b. $65,738.04
c. 18%

4. NPV = PV(Benefits) – PV (costs). 6,900 = PV(Benefits) – 15000. Therefore PV


(Benefits) = $21,900.

Use cash flow keys to find IRR = 24.95%.

PI = PV (benefits)/ PV(costs) = 21900/15000 = 1.46

Cost of capital (or discount rate). Since we know that the present value of the benefits equals
$21,900, we can solve for the discount rate.
PMT = 4500; N = 8; PV = -21,900; FV = 0; CPT I/Y. I/Y = cost of capital = 12.59%.

5. PI = PV (Benefits)/ PV (Costs). Thus we have 0.96 = PV (Benefits)/1000000. So,


PV(Benefits) = $960,000.
NPV = PV(Benefits) – PV(Costs) = 960000 – 1000000 = -$40,000.

6. b.

7. e

8. a

9. NPV = $9,298.81; IRR = 19.63%; PI = 1.116. Accept Project as NPV is


positive.

10. B. It has the highest NPV.

11. A,B,C,D (ALL). They all have positive NPV.

12. NPV = $54.85

13. Bonds: PV = -1095; FV = 1000; PMT = 116; N =24; CPT I/Y. I/Y = Pretax cost of
debt = 10.50%. After-tax cost of debt = (1-0.3) (10.5) = 7.35%

Stock. k = 0.07+5.25(1.07)/68.25 = 15.23%

Cost of capital = 0.5(7.35) + 0.5(15.23) = 11.29%

14. Bonds: PV = -8785; N = 60; FV = 10000; PMT = 390; CPT I/Y. I/Y = 4.487%. Thus
the YTM = Pre-tax cost of debt = 2(4.487) = 8.97%.
After-tax cost of debt = (1-0.30)(8.97%) = 6.28%.

Stocks: 33.75 = 2.90(1.0375)/(k-0.0375). Solving for k we get k = 12.66%, the cost of


equity.

Thus, cost of capital = 0.8(6.28) + 0.2 (12.66) = 7.56%.


CHAPTER 11
Assignment 11.1:
1.      Determine Net Cash Flow: NCF = (2,000,000 – Depreciation)(1 – T) + Depreciation
NCF = (2,000,000 – 1,000,000)(.7) + 1,000,000 = 1,700,000.
CF0 = -10000000; C01= 1700000; F01=10; Compute NPV (with I = 13) = -775,386.09.

2.      Project A: CF0 = -80000; C01=18000; F01=8; Compute NPV (with I = 10) = 16,028.67. With same
figures in CF register, compute IRR = 15.29%

Project B: CF0 = -40000; C01=10000; F01=8; Compute NPV (with I = 10) = 13,349.26. With same
figures in CF register, compute IRR = 18.62%

Note that NPV indicates that we should accept Project A and IRR indicates that we should accept
Project B. ALWAYS CHOOSE THE PROJECT WITH THE HIGHEST NPV, so choose Project A.

3.      Enter all values in cash flow register and compute:

NPV (A) = 4497.54; IRR(A) = 19.31%


NPV (B) = 7937.38; IRR(B) = 28.97%
 Choose Project B because higher NPV.

4.      If you construct graph correctly, you should find that the two lines cross at a discount rate of
approximately 16.25%. Note that with I = 16.25, NPV (P) = 1249.76 and NPV (Q) = 1249.77.
 At rates above 16.25%, NPV(P) > NPV(Q)
 At rates above 16.25%, NPV(P) < NPV(Q).

Assignment 11.2:
1.      (Assume tax rate = 0). First compute the NPV of cost of each machine.
X100: CF0=-45000; C01=-5000; F01=4; C02=-5000+6000=1000; F02=1; Compute NPV (I=12) =
-59619.32. Note that this is NPV of costs.

X1300: CF0=-65000; C01=-2000; F01=6; C02=-2000+10000=8000; F02=1; Compute NPV (I=12) =


-69604.02. Note that this is NPV of costs.

Because these two projects have different lives, to compare we must now compute EAC.
EAC(X100): PV = -59619.32; N=5 (i.e., project life); I/Y=12, CPT PMT = 16,538.98.
EAC(X1300): PV = -69604.02; N=7 (i.e., project life); I/Y=12, CPT PMT = 15,251.48.

Thus EAC (X100) = 16,538.98. EAC (X1300) = 15,251.48. Choose X1300 because it has a lower
equivalent annual cost.
2.      Initial Cash Outflow:
Buy Machine -31M
Decrease in NWC + 2M
CF0 -29M

Note that annual depreciation = (31-3)/20 = 1.4M

Annual Operating NCF = (Rev – Costs – Dep)(1-T) + Dep


= [(5)(.9) – (.6)(5)(.9) - .6 – 1.4](.7) + 1.4 = 1.26M per year for 20 years

Final Year Non-Operating Cash Flow:


Sell Machine for Scrap Value 3M
Replace NWC -2M
Total 1M

To Compute NPV: CF0 = -29, C01 = 1.26; F01 = 19; C02 = 2.26; F02 = 1;
CPT NPV (with I = 11) = -18.842M

Assignment 11.3:
Q36: CF0 = -3M; C01 = .6M; F01 = 8; CPT NPV = 200,955.72.
Z96: CF0 = -4M; C01 = .7M; F01 = 10; CPT NPV = 301,196.97.

Since these projects have different lives, to compare, must compute Equivalent Annual Annuity.

EAA(Q36): PV = 200955.72; I = 10; N = 8; CPT PMT = EAA = 37,667.95.


EAA(Z96): PV = 301196.97; I = 10; N = 10; CPT PMT = EAA = 49018.42.

Choose Z96, since it has higher EAA.

Problems:
1.      Must compute EAA of each project. Rank of EAA: D, C, A, B.
2.     
 Relevant – Opportunity cost
 Not relevant – Sunk cost
 Relevant – Initial cost
 Not relevant – Sunk cost
 Relevant – Incremental expense
 Not relevant – Sunk cost
 The incremental revenue of 93% of $2 million is relevant.
 The tax savings associated with depreciation is relevant. Specifically, 36% of incremental
depreciation.
 Relevant – Incremental expense
 Not relevant – No incremental cost
 Relevant – Incremental
 Not relevant – must be paid regardless of whether project is accepted or rejected. Therefore, not
incremental.
3.      The easiest way to do this problem is to assume the company can sell radiators for $150 per
radiator. Then:
NPV (Buy from Supplier) = 13,477,081. NPV (Manufacture) = 9,439,730. ???
Project Buy: CF0=0, CF1=(150-120)*100,00*(1-0.3)=2.1 million, F1=10, NPV=13,477,081

Project Manufacture:

(150-80)*100,000=7 mil , 7 mil – 0.5m (depreciation)=6.5 million, 6.5*(1-0.3)=4.55 mil


4.55 mil + 0.5 mil (depreciation)=5.05 mil

CF0=-5 million, CF1=5.05 mil, F1=10, NPV= 27,409,171

So make the radiators.

4.      EAC(A) = 1160.76; EAC(B) = 1813.90 – Choose project with lower equivalent annual cost.
5.      NPV(Superior) = 17,802.68; NPV (Peerless) = 16,816.12 – Choose project with highest NPV.
6.      IRR(A) = 34.90%; NPV(A) = 5,849.33
IRR(B) = 31.61%; NPV(B) = 10,490.40
7.      Crossover rate = 7.167%; NPV(X) = NPV(Y) = 6720.60.
8.      a. Yes – incremental; b. No – Sunk; c. No – financing cost (captured in NPV analysis);
d. Yes – incremental; e. Yes – incremental.
9.      Note that annual NCF = Net income + Depreciation = 700,000 per year. NPV = 227,462.17.
10.  3,690.93
11.  Note that annual NCF = Net income + Depreciation = 6.55M per year. NPV = -5.879M.
12.  CF0 = (12,000,000); CF1 through CF10 = 2,880,000; No final year non-operating cash flow. NPV =
1,919,695.
13.  CF0 = (26,800,000); Operating net cash flow = 26,800,000; Final year non-operating cash flow =
2,200,000. NPV = 97,762,829.
14.  (56,004.33)
15.  For this problem, assume that the cost of capital is 14%. Then, NPV = (320,096). Reject.
16. 
Step 1: Initial Investment in the Project
 
Opportunity cost of the land           -8M
Buy knives, blades, etc.                -5.7M
New Building construction             -70M
Increase in Net Working capital     -10M
   Total Year 0 Cash Flow             -93.7M
 
Step 2: Annual Operating Cash Flow
Note that Incremental revenue = (3000)(300)(30) = 27M
and Depreciation = (5.7-.7)/25 + 70/25 = 3.0M
 
NCF = (27M - 3.2M - 4.5M - 2.3M - 3.0M)(.6) + 3.0M = 11.4M for 25 years.
 
Step 3: Final Year non-operating Cash Flow
Sell land                       8M
Sell equipment             .7M
Recover NWC             10M
   Total                       18.7M
 
To compute NPV:
CF0=-93.7; C01 = 11.4; F01=24; C02=11.4+18.7=30.1; F02 = 1; CPT NPV = -14.642M.

You might also like